« first day (4501 days earlier)      last day (506 days later) » 
00:00 - 21:0021:00 - 00:00

12:33 AM
@TedShifrin Thanks Ted! It makes sense. I shouldn't be afraid of going to coordinates in the future. Lesson learned.
 
If $|Li(x)-h(x)|=\frac{1}{2}|Li(x)-\pi(x)|$ and $|Li(x)-\pi(x)|=O(xe^{-c \sqrt{\log x}})$ then what does that say about the error term for the former?
I think they would have the same error term because it's just a constant
 
@geocalc33 Correct, because big O notation ignores constant multipliers.
@geocalc33 What are the two blue curves?
@KZ-Spectra That's weird. I just tested it in the Answer editor and it looks correct in the preview.
 
@PM2Ring def theta(x): return sum(math.exp((log(p))^-1) for p in prime_range(1,floor(x)+1))
def psi(x): return sum(math.exp((log(1/p))^-1) for p in prime_range(1,floor(x)+1))
so basically it's useless because it just about halves the differences and doesn't effect the big O
 
12:49 AM
@geocalc33 Oh well. But as Semiclassical mentioned, it's not easy to improve on the big O because pi(x) & Li(x) have infinite crossover points.
@geocalc33 No wonder it times out, though. Those functions have to recalculate the sum for every x value. Let me see if I can make a faster version...
 
I'm not actually sure about that 1/2 factor
only plotted up to 2500
 
1:35 AM
@geocalc33 Here's a demo. We use a for loop to put the cumulative sums into a list. We could compute both sums in the same loop. This code takes a second or two to plot up to m=100000. Don't forget to comment out the print call. ;)
 
1:48 AM
Here's psi & theta.
Upto a million takes ~15 seconds.
 
got the average
won't let me use prime_pi(x) anymore as a function in this code
 
@geocalc33 Just give it the function name, without the x. [psi, theta, prime_pi]
Actually, it should work with the x too. Really, we should specify x in the plotting domain: (x, 0, m)
Maybe you've over-written the built-in symbolic x variable.
 
got it to work thanks
 
Oh, good. :)
 
2:07 AM
found something cool
the differences |Li(x)-pi(x)| stay lower over larger magnitudes
 
@geocalc33 Maybe. ;) It's dangerous to make generalisations about that. Just ask Littlewood.
Did you see the stuff Robjohn & I did back in August on the number of primes in the intervals $[n^2-n, n^2+n]$ ? chat.stackexchange.com/transcript/36?m=61882200#61882200
Jul 4, 2021 at 20:58, by PM 2Ring
Speaking of prime random walks, here's an image I just made from the residues mod 5 of the primes <10,000,000. Start at (0, 0). Then if the next residue is 1 increase x, if it's 4 decrease x, if it's 2 increase y, if it's 3 decrease y. Count the times you visit each point, and convert that to a color. I do a log(1 + k) transform (where k is the visit count) to make the pattern more visible.
 
What is a good multiple shooting solver for BVP? Is there one you would recommend other than Matlab and Mathematica?
 
2:24 AM
@Brownian_Motion You could use Sage, I guess. I've used it for a few IVPs, but not BVPs. doc.sagemath.org/html/en/reference/calculus/sage/calculus/…
geocalc33, also:
Jul 2, 2021 at 11:46, by PM 2Ring
The recent kerfuffle about the Riemann hypothesis inspired me to write a little Sage / Python script to explore the graph of the cumulative Liouville function, A002819. There are a couple of graphs on Wikipedia, but they're pretty crude, and you can't zoom in.
 
2:39 AM
woahh
 
en.wikipedia.org/wiki/Skewes%27s_number has info on the Li(x) , pi(x) crossovers.
 
subtracting the green from the red gives the same behavior as the absolute differences between pi and Li
 
They certainly look close. But your function names are a bit cryptic.
 
|rho(x)-pi(x)|-|Li(x)-rho(x)| is essentially the same as |Li(x)-pi(x)|
is what the plot is telling me
where rho(x) is that averaged function in the plot^^^ up there
does that just trivially simplify?
no because then the blue plots would exactly coincide
 
3:07 AM
Maybe something can be done using exp(1 / log(p)) + exp(-1 / log(p)) = 2cosh(1 / log(p))
 
3:17 AM
If $f(x)\sim g(x)$ as $x\to\infty$ then $\int f(x)\sim \int g(x)$ as $x\to\infty$?
 
Way vague.
 
Let $f,g:\Bbb R\to\Bbb R$ be nonnegative increasing (not necessarily strict) functions such that $f(x)\sim g(x)$ as $x\to\infty$. Then $\int_1^xf(t)\ dt\sim\int_1^xg(t)\ dt$ as $x\to\infty$?
Oh there is a known result that if one of the integral diverges then the statement holds.
 
And $\sim$ means precisely?
 
3:36 AM
comme ci, comme ça
 
@TedShifrin $f(x)\sim g(x)$ as $x\to\infty$ if $\lim_{x\to\infty}{f(x)\over g(x)} =1$.
Actually the problem is from complex analysis which is: if $\psi(x) = \sum_{p\leq x,p:\text{prime}}\lfloor {\log x\over\log p}\rfloor \log p$ and $\psi(x)\sim x$ as $x\to\infty$ then $\int_1^x\psi(u)\ du\sim{x^2/2}$ as $x\to\infty$.
 
@copper.hat Exactement!
 
One minor thing is that $\psi$ is not continuous but maybe I can approximate by a continuous function
The problem is from stein CA but no complex analysis in this problem so I don't like this that much
 
Well, ok, it seems now it follows from L’Höpital, no?
This is number theory stuff, so it can’t be all complex analysis.
 
Suppose that $\mu, \nu$ are finite measures on (X,F). Given $f=\frac{d\nu}{d(\nu+\mu)}$, prove that $\nu(E)= \int_E \frac f{1-f} d(\nu+\mu)$.
How is this possible?
 
3:42 AM
I guess I’m assuming continuity when I say L’H.
The denominator seems wrong, eh, Koro.
 
So you're saying $F(x) = \int_1^x f(t)\ dt, G(t) = \int_1^xg(t)\ dt$ then by LH
$${F(x)\over G(x)} = {f(x)\over g(x)}$$
and RHS $\to 1$ as $x\to\infty$ assumping $F,G\to\infty$ as $x\to\infty$.
 
Oh, i should have said $\nu (E)=\int_E \frac f{1-f} d\color{red}{\mu}$
It is true that $\mu<<\mu+\nu$ so there exists Radon Nikodym derivative $h$ so that $\mu(E)= \int h d(\mu+\nu)$. It is given that $\nu (E)=\int_E f d(\mu+\nu)$. Adding the two gives: $\mu+\nu(E)=\int_E (f+h) d(\mu+\nu)$ So by uniqueness of the Radon Nikodym derivative, I get $f+h= 1$ almost everywhere.
But not sure how to go from here.
 
You need limits, potato.
 
Ted: the denominator was correct. Inside of d(...) was not correct at one place. Fixed now.
 
Now it works.
 
3:49 AM
But anyway need continuity of $f$ and $g$.
 
Test $a\left\{\begin{align}x,&x\ge0\\-x,&x<0\end{align}\right}+b$
 
$d\mu/d\nu = 1/f - 1$, modulo technicalities.
Invert and you get it.
 
yeah, but how did you get it?
I also want to do $\frac{d\mu}{d\nu}= \frac{\frac{d\mu}{d(\mu+\nu)}}{\frac{d\nu}{d(\mu+\nu)}}$
But not sure if it is allowed.
 
So, if the result is true, we must have $\nu <<\mu$?
Is everyone a.c. W.r. To everyone?
 
that's the problem. It is not even given that \nu is ac. w.r.t. \mu.
But even if we assume that(and we must else the question makes no sense), still is cancellation of denominator is allowed?
 
4:15 AM
10
A: Radon-Nikodym Theorem for (positive) measures, chain rule

user95849First, we prove a lemma: Lemma: Let $(X, M, \mu)$ be a measure space. Let $f$ be a nonnegative measurable function on $(X, M)$. Define a measure $\nu$ on $(X, M)$ by $\nu(E) = \int_{E}fd\mu$ for $E \in M$. Then for any nonnegative, measurable function $F$ on $(X, M)$, we have $$\int_{X}Fd\nu = ...

How is this non-trivial?
How can this be put into an assignment problem?
Measure theory is my least favorite now.
But the linked answer is very nice :-).
 
5:00 AM
If $p:\Bbb R^{n+1}\setminus\{0\}\to S^n$ is a projection map $x\mapsto x/||x||$ and $\omega$ is a closed $n$-form on $\Bbb R^{n+1}\setminus\{0\}$ then is are any $\eta\in\Omega^n(S^n)$ such that $p^*\eta = \omega$?
 
5:20 AM
If f is an even function, then why is $\int_0^\infty \int_{S^{n-1}} f(x) r^{n-1} d\sigma dr$ equal to $\sigma(S^{n-1})\int_0^\infty r^{n-1} dr$?
 
@onepotatotwopotato Not necessarily. What property must a pullback form have?
@Koro certainly not true.
 
f is an even function.
It is a corollary in Folland's.
f(x)=f(-x)=f(|x|)
Ahh, $\int_0^\infty \int_{S^{n-1}} f(\color{red}{r}x) r^{n-1} d\sigma dr$
 
@TedShifrin The actual question is different. If $\omega\in\Omega^n(\Bbb R^{n+1}\setminus\{0\})$ is a closed form and $M$ is a compact orientable manifold without boundary with a smooth map $f:M\to\Bbb R^{n+1}\setminus\{0\}$ then $\int_Mf^*\omega$ is an integer multiple of $\int_{S^n}\iota^*\omega$. If $p$ is the projection map I defined, then $\int_M(p\circ f)^*\omega = k\int_{S^n}(p\circ\iota)^*\omega$ for some $k\in\Bbb Z$ where $\iota$ is an inclusion map $S^n\to\Bbb R^{n+1}\setminus\{0\}$
 
Still not right, but closer.
 
So I hoped there is $\eta\in\Omega^n(S^n)$ that $p^*\eta = \omega$ but well no.
 
5:34 AM
This is very different. This is like winding number,
 
no, I am wrong. even odd don't make sense in R^n
 
Yes, they do.
 
Ted: my hypothesis was wrong.
 
But, regardless, the integral depends on $f$.
 
and the expression as well.
One should say that f(x)= g(|x|), f is from R^n to R. So the integral I wrote simply becomes $\sigma (S^{n-1}) \int_0^\infty \color{red}{g(r)} r^{n-1} dr$.
 
5:38 AM
OK
 
5:59 AM
There was a question in the exam, Using the CLT, show that $\lim\sqrt n P(X=n/2)=\sqrt{2/\pi}$ where $X$ is the number of heads when we flip a coin $n$ times. They also gave a hint that for a continuous function $\int_{-a}^a f(x)\sim 2af(x)$.
Here $n$ is even.
So the thing is that you have to write $P(X=n/2)=P(\frac{n-1}2<X<\frac{n+1}2)$ then you'll get the $\sqrt{2/\pi}$
What I did was $P(X=n/2)=P(\frac{n}2-\epsilon<X<\frac{n}2+\epsilon)$ where $\epsilon$ is a small real number $<1$. Then I found the end result in the form of $\epsilon$.
So changing the epsilon values we'll get different limits so limit shouldn't exist.
Why do we only take $\frac12$? I have seen other problems where they use $\epsilon=\frac12$.
Since binomial r.v. pmf is only concentrated at integers so it shouldn't matter what epsilon we take. RIght?
 
6:19 AM
@TedShifrin Wait. $H^n_{dR}(S^n)\simeq H^n_{dR}(\Bbb R^{n+1}\setminus\{0\})$ via $p$.
$\omega$ is a closed form
 
6:40 AM
@onepotatotwopotato Yes, of course they’re isomorphic. That doesn’t mean every closed form is a pullback.
 
7:19 AM
One (surprising?) thing I found: If $f:\Bbb C\to\Bbb C$ is a polynomial then we can extend this map to a continuous map $\bar{f}:\Bbb C_\infty\to\Bbb C_{\infty}$. The degree of $\bar{f}$ in a topological sense is equal to the degree of $f$ (polynomial degree).
 
 
2 hours later…
9:45 AM
I suspect I have found a typo in Spivak’s Calculus, third edition, on page 144. He writes “Then the Lemma implies that $f$ is $\epsilon$-good on $[a, a+\delta_0]$, so $a+\delta_0$ is in $A$…” I suspect he means $\alpha+\delta_0$ instead of $ a+\delta_0$.
 
 
2 hours later…
12:10 PM
@onepotatotwopotato Yes, it is known (and should appear in textbooks) that polynomials are meromorphic functions of the Riemann sphere, and the degree coincides with the index at infinity.
 
@Yai0Phah It was an exam problem. I didn't know it before.
 
@TedShifrin L'Hôpital, a French name.
 
L'Hopital or L'Hospital?
 
neither of these.
 
12:37 PM
@onepotatotwopotato this leads to a classic proof of the funndamental theorem of algebra
 
If $X,Y$ are two nonvanishing vector fields on $M$ (smooth mfd) then $X = fY$ for some smooth function $f\in C^\infty(M)$?
 
consider $M=\mathbb{R}^n$
 
Ah ok.
Then I would say $M$ is 1-dimensional.
Oh I can just divide the component function.
 
1:03 PM
then sure
 
1:19 PM
generally, any two nowhere vanishing sections of a 1-dimensional line bundle over a smooth mainfold are multiples of one another by smooth functions
 
1:58 PM
@LukasHeger yo Lukas , can I send you my homework to check very fast if it is good or not? it is 3 questions
I always have second thought on how it looks written on latex
everything is done just a quick look to boost my confidence in writing good mathematics !
 
@JackOhara Interestingly, i'm teaching a course in LaTeX right now.
 
@Ajay wow lol is it online?
 
No, I teach a group of about 45 students and 6 teachers.
 
or do you have Youtube chanel?
ah darn :/
 
I have some resources I can share with you if you want.
Most of the stuff is online.
 
2:01 PM
Yes please!
 
Just not publicly
K, can I get ur email.
 
Yes ofcourse
 
Ok, give me some times, I need to go to school tmr and log onto my school laptop, will share then.
 
Okay but did you copy the email?
 
Yes.
 
2:04 PM
OK ! :)
rather not have in public for long haha
 
True lol.
I've been having some fun recently
 
with ? =)
 
Stuff like this
I love messing with children.
 
haha
 
I've been creating loads of gifs.
 
2:06 PM
wait you teaching latex to kids?
as in highschool?
or you just doing this extra for fun ?
 
Yeas, and teachers.
 
ah great
 
No it's a seven week program the head of the amths department asked me to do.
And kids are used in the sense that they don't have much "higher" math exposure.
 
@Jakobian are you around? I'm trying to understand a proof that looks like it might be your cup of tea
 
Like for Ted and Leslie, we are toddlers.
 
2:07 PM
ah I see =)
 
Anyways, good night.
 
good night!
how to prove that the closed interval [a,b] in R is connected?
without referring to topology
I can use the completness axiom of R
and this theorem that says , X a subset of R is connected then for any element z in X in such that a<z<b , then z has to be in X
 
I just found the proof of FTA using Stoke's theorem
 
I tried to write an argument by contradiction
 
2:31 PM
@AlessandroCodenotti I'm on a lecture but go on
 
3:09 PM
@Jakobian $X$ is a compact space, $C_p(X)$ is the space of continuous real valued functions $X\to\Bbb R$ with the pointwise convergence topology. I want to show that if $Y\subseteq C_p(X)$ is closed, then $Y$ is compact iff it is countably compact
This is a theorem of Grothendieck
 
Seems like a mundane theorem to have Grothendieck's name attached to it. Is it super important, @AlessandroCodenotti?
 
It's fairly important in the study of compact subsets of function spaces and in the study of Rosenthal compacta. The main result is the Bourgain-Fremling-Talagrand dychotomy though
 
@AlessandroCodenotti yeah I know of this theorem
 
I don't understand a step in the proof, but it's toward the end, do you have time and are you interested in quickly going through the proof? (It's not long, less than a page in the book I'm looking at)
 
The proof is a bit technical and goes by contradiction
Sure
 
3:24 PM
Alright. We assume that $Y$ is countably compact and we want to show that it is compact. Since countably compact means that every sequence has an accumulation point we can find, for every $x\in X$ a constant $M_x$ such that $|f(x)|\leq M_x$ for all $f\in Y$. This shows that $\overline{Y}$ (closure taken in $\Bbb R^X$) is compact, since it is closed in $\prod_{x\in X}[-M_x,M_x]$, so we have reduced to showing that in fact $\overline{Y}\subseteq C_p(X)$.
Suppose for a contradiction that there is a discontinuous $f\in\overline{Y}$. We can find $\varepsilon>0$ and $y\in X$ such that $Z=X\setminus f^{-1}(f(y)-\varepsilon,f(y)+\varepsilon)$ accumulates to $y$
Now this step is not 100% clear to me, but it is reasonable. Since $f$ is discontinuous we can find some $r\in\Bbb R$ and $\varepsilon>0$ such that $f^{-1}(B_\varepsilon(r))$ is not open, so $X\setminus f^{-1}(B_\varepsilon(r))$ is not closed, so it must accumulate to some point in its complement
But it's not clear to me how we get the same $y$ to be the center of the ball and the accumulation point at the same time
But anyway this doesn't seem crucial in the following so let's ignore this issue for a second
 
@AlessandroCodenotti Maybe I am missing something, but the above just seems to say that "f is not continuous at the point y".
Sorry for the interruption - you said that this isn't the most relevant iissue here.
 
Oh right of course
We're just saying that there is a sequence of points that accumulates to $y$ but which $f$ maps $\varepsilon$ away from it
Thanks
 
Maybe not necessarily a sequence - let's say a net.
But I guess that should be the gist of the part you wrote above.
 
Alright so now the idea is to construct sequences of open nbhds of $y$, $\{U_n\}$, $\{x_n\}\subseteq Z$ and $\{f_n\}\subseteq Y$ such that, for all $n$:
0) $\overline{U_{n+1}}\subseteq U_n$
1) $|f_n(x)-f_n(y)|<\varepsilon/2^n$ for all $x\in U_n$
2) $x_n\in U_n\cap Z$ (that is $|f(x_n)-f(y)|\geq\varepsilon$
3) $|f_{n+1}(x_i)-f(y)|>\varepsilon/2$ for $i=1,\ldots,n$
4) $|f_n(y)-f(y)|<\varepsilon/2^n$
Here again let's just assume we have such sequences, this is not the issue
Now we let $x_\infty$ be any accumulation point of $\{x_n\}$, so in particular $x_\infty\in\bigcap\overline{U_i}$, and we let $S=\{x_n\}\cup\{x_\infty\}$. The restriction map $\Phi\colon C_p(X)\to C_p(S)$ is continuous, in particular $F=\Phi(Y)\subseteq C_p(Y)$ is countably compact (being the image of a countably compact set) and so it must actually be compact, since $\Bbb R^S$ is separable metric and countable compactness is the same as compactness for such spaces
Since $F$ is compact, we can find $g\in F\subseteq C_p(S)$ at which $\{f_n\}$ accumulates. But now we show that $g$ is discontinuous, giving the desired contradiction.
The book here just says that $g$ is discontinuous at $x_\infty$, since by construction $g(x_\infty)$ is not in the closure of $\{g(x_n)\}$, but I could definitely use some more details since I'm not seeing why is that the case
 
3:54 PM
I'll let you know what I think later since reading this on phone isn't the best
 
Sure, there's no hurry
The reference I'm reading is Todorcevic's topics in topology, section 1, if you want to look at that directly
 
BTW the proof can be found as U.044 in Tkachuk: A Cp-Theory Problem Book.
I am unable to translate between the two notations quickly enough - but if nothing else, the final part of the proof (where continuity of $g$ leads to a contradiction) seems to be more detailed.
 
Ah thanks, I'll take a look there as well
 
4:13 PM
Cannot we get $|g(x_\infty)-f(y)|\ge\varepsilon/2$ from 3?
 
Hmm because $g$ is an accumulation point of the $f_n$, so it will be very close to big enough $f_k$ on $x_i$
But what is the point of all the other conditions?
 
I was thinking about using 1) and 4) to get $|f_n(x)-f(y)|<\varepsilon/2^{n-1}$ for $x\in U_n$.
 
I'd agree with Peter this years were worst year
 
Consequently $|g(x)-f(y)|\le\varepsilon/2^{n-1}$ for $x\in U_n$. And then use $(\forall n) x_\infty\in U_n$ to get $g(x_\infty)=f(y)$.
As you rightly said, this seems rather suspicious. If that was correct, I did not use 2) at all.
Or maybe such argument would use 2) - in that we're using $x_\infty \in \bigcap \overline U_i$...?
I think I managed to confuse myself a bit....
Sorry, I'll have to leave. I am not sure if this was any help at all.
 
4:39 PM
I'm also busy at the moment, I'm not thinking carefully about it, I'll check later/tomorrow
 
5:29 PM
@onepotatotwopotato You should look at my book more often :D
 
6:09 PM
Hi everyone! I am trying to derive a formula for the 7-rough numbers, which are numbers relatively prime to 2, 3 and 5, or having only prime factors >= 7. I'd like to follow a reasoning similar to the one here (math.stackexchange.com/a/928114/719906), but I cannot make any sense out of the 7-rough sequence.
For sure the differences between adjacent numbers repeat with a pattern: [6, 4, 2, 4, 2, 4, 6, 2] repeated. Does this mean I should take the average value plus a constant as for the numbers relatively prime to 2 and 3? Or maybe a quadratic expression? And how should additional terms be included (based on odd or even, or other parameters I am not aware of)?
By the way, this is how the sequence of the 7-rough numbers looks like: 1, 7, 11, 13, 17, 19, 23, 29, 31, 37, 41, 43, 47, 49, 53, 59, 61, 67, 71, 73, 77, 79, 83,...
Do you have any hints regarding this?
 
I should take a look too. I never had things like Stokes theorem derived in my courses
@AlessandroCodenotti you mean that $y$ is $Z$'s accumulation point?
I don't think I ever heard of "accumulates" before in this context
 
6:28 PM
@Jakobian yes
 
7:09 PM
$g(x_\infty) = g(y) = f(y)$ from 1) and 4)
3) gives $|g(x_n)-f(y)| > \varepsilon/2$ though
@AlessandroCodenotti I don't think I used 2) anywhere though, but maybe it's a condition to help the recursion going
 
Yeah maybe 2 is uses in the recursive step, I'll check carefuly tomorrow
 
7:30 PM
Could maybe someone help me here in functional analysis:
This would be my question:
Let $X$ be a Banach space, $T\in B(X,X)=\{T:X\rightarrow X|~ T~\text{is linear and bounded operator}\}$ and denote by $\sigma(T):=\{\lambda \in \Bbb{C}: (\lambda I-T) ~\text{is not invertible}\}$ the spectrum of $T$.

Consider the map: $$\phi: \Bbb{C}\setminus \sigma(T)\rightarrow B(X,X): \lambda \mapsto (\lambda I-T)^{-1}$$
I want to check that $\phi$ is analytic.
We did the following proof:

>Let $\lambda_0\in \Bbb{C}\setminus \sigma(T)$, then $$\begin{align} (\lambda I-T)&=(\lambda-\lambda_0)I+(\lambda_0 I-T)\stackrel{(\lambda_0 I-T)~\text{is invertible}}{=}(\lambda_0I-T)((\lambda-\lamb
 
@user123234 (1) is something that in every Banach algebra, it follows from the usual geometric sum formula
or to put it differently, what happens if you multiply $\sum_{n=0}^\infty (-1)^n S^n$ by $(I+S)$?
note that this sum converges absolutely
 
but shouldn't it be $\sum_{n=0}^\infty S^n$?
 
that would be for $(I-S)^{-1}$
 
$\frac{1}{1-x} = \sum_{n=0}^\infty x^n$
 
ah sorry I see. But then there is still one question, why can I "naivly" apply the geometric sum formula in any banach space?
 
7:40 PM
*banach algebra
thes set of bounded operators is a banach algebra
 
I have never heard about this.
so about a banach algebra
 
Fix $r>0$, consider $A_r$ of complex power series in one variable with convergence radius at least $r$ around $0$, we can think of those as the holomorphic functions from set $U_r:=\{z \in \Bbb C \mid |z|<r\}$ to $\Bbb C$. This is a $\Bbb C$-algebra. If $A$ is any banach algebra and $S \in A$ with $\|S\|<r$, then we get a ring homomorphism $A_r \to A, f \mapsto f(S)$
 
now in $A_r$ we have the equation $(1+z)^{-1}=\sum_{n=0}(-1)^nz^n$
and then we can just apply the ring homomorphism to that
you can do this for some $r$ with $\|S\|<r<1$
 
ah and since it is a homomorphism it maps $1\mapsto I$ and $z\mapsto S$?
 
7:52 PM
yes
and it preserves inverses as well (every ring homomorphism does)
technically we don't know if it preserves infinite sums, but the definition of $f(S)$ is just that you plug $S$ into the power series
 
I got it. So the crucial argument here is that we have a ring homomorphism between the space of complex power series and $A$?
 
yeah basically
 
but what I still do not see is the fact that $A$ is a banach algebra, in our case is A=X?
 
yeah, that's the best way of thinking about it.
 
in our case $A=B(X,X)$
 
7:55 PM
the low tech way of thinking about it in the case of this series is that all of the usual theorems for manipulating series still work (and in a banach algebra with much the same proofs).
 
just beware when you have multiple elements in a Banach algebra that don't commute: in general $e^{S+T} \neq e^{S}e^{T}$
but all single-variable identities between power series carry over
 
but I don't get why B(X,X) needs to be a banach algebra for this, and firstly what is a banach algebra? Is it a set B(X,X) with a natural map $X\rightarrow B(X,X)$?
 
ANT gave me a trauma regarding "formal power series" arguments
 
@user123234 do you know that $B(X,Y)$ is a banach space for banach spaces $X,Y$ (technically only $Y$ needs to be banach, $Y$ can be just normed)
you have the operator norm
 
Yes I know this
 
7:58 PM
If $f:G\rightarrow G$ is a injective Lie group homomorphism where $G$ is compact, does it follows that $f$ is surjective?
 
okay so we know that $B(X,X)$ is a banach space with the operator norm. We also have multiplication, just given by composition. We know that the operator norm satisfies $\|f \circ g \| \leq \|f\|\|g\|$, so we have a Banach algebra
 
yes if you assume $G$ connected
 
Yes. That I know
i'm wondering about if $G$ is compact
 
ah okey so the general definition of a banach algebra is :

B is a banach algebra if B is a banach space, multiplication is defined and the norm is submultiplicative?
 
yes
 
8:01 PM
many of those series manipulation arguments are implicitly using the cauchy criterion, which is why completeness is often assumed (and needed for many arguments to work)
 
yes, I assumed that $G$ is compact
 
perfect thanks I got this!
 
I mean compactness without connectedness
 
actually, yeah, you don't need connected
 
8:03 PM
So we know that $G^0 \to G^0$ is bijective, so we get an injection $\pi_0(G) =G/G^0 \to \pi_0(G)$, but $\pi_0(G)$ is finite
 
an injective Lie group homorphism $G\rightarrow H$ between Lie groups of the same dimension with $G$ compact is always an embedding with clopen image, i.e. an isomorphism onto a collection of components of $H$. if $G=H$, the number of components is finite and since it's a homeomorphism invariant, it has to be all.
yeah, Lukas put it more elegantly
 
Hi everyone. I'm new to projective special linear groups. How come $PSL(2,3)\cong A_4$?
I can verify that it is in GAP. That's as far as I've got.
 
For a lie group $G$, always $\pi_0(G)= G/G^0$?
 
Where are my manners? Please help :)
 
8:19 PM
@Shaun You can certainly verify with generators and relations, but intuitively it's the automorphism group of $\Bbb P^1(\Bbb Z_3)$, which has 4 points.
I guess you should stop to verify that there are no transpositions in there.
@monoidal Yes, it's the number of connected components.
Ah, MSE is malfunctioning again. It's not wanting to post comments.
 
@TedShifrin Thank you. I'm looking for something more elementary than that though. I should have said so. I'm sorry. Here $PSL(n,F)$ is defined as the quotient of $SL(n,F)$ by its centre.
 
Yes, I know that.
If you're not going to think about the projective geometry, then just work out generators and relations.
You should probably understand the basic situation with $\Bbb P^1(\Bbb C)$ and linear fractional transformations up to scalars (which again will be $PSL(2,\Bbb C)$).
 
@TedShifrin Okay. I'll try that and then, if I get stuck, I'll ask a question on the main site. Thank you.
 
8:37 PM
@TedShifrin you can also show that $PGL(2,3) \cong S_4$ (via the action on $\Bbb P^1(\Bbb F_3)$, that's pretty direct) and then argue that $S_4$ has a unique subgroup of index 2
but the projective geometry really isn't harder than some basic linear algebra: $\Bbb P^1(\Bbb F_3)$ is the set of 1-d subspaces of $\Bbb F_3^2$
you don't even need homogenous coordinates for this I think
if $L$ is a one-dimensional subspace of $\Bbb F_3^2$ and $A \in \mathrm{GL}_2(\Bbb F_3)$, then $AL$ is again a one-dimenional subspace
this gives you an action of $\mathrm{GL}(2,3)$ on $\Bbb P^1(\Bbb F_3)$
and then you need to show that it factors over the center
oops I meant to ping @Shaun
I wonder if there's a proof using a tetrahedron
 
@LukasHeger That's more in line with what I was looking for. Thank you. Again, I'll think it through in the meantime then ask for help later if needs be.
 
00:00 - 21:0021:00 - 00:00

« first day (4501 days earlier)      last day (506 days later) »